You are on page 1of 23

PHYS 110B - HW #4

Fall 2005, Solutions by David Pace


Equations referenced as ”EQ. #” are from Griffiths
Problem statements are paraphrased

[1.] Problem 8.2 from Griffiths


Reference problem 7.31 (figure 7.43).
(a) Let the charge on the ends of the wire be zero at t = 0. Find the electric and magnetic fields in
~ t) and B(s,
the gap, E(s, ~ t).

(b) Find the energy density and Poynting vector in the gap. Verify equation 8.14.

(c) Solve for the total energy in the gap; it will be time-dependent. Find the total power flowing into
the gap by integrating the Poynting vector over the relevant surface. Verify that the input power is
equivalent to the rate of increasing energy in the gap. (Griffiths Hint: This verification amounts to
proving the validity of equation 8.9 in the case where W = 0.)

Solution
(a) The electric field between the plates of a parallel plate capacitor is known to be (see example 2.5
in Griffiths),
~ = σ ẑ
E (1)
o
where I define ẑ as the direction in which the current is flowing.

We may assume that the charge is always spread uniformly over the surfaces of the wire. The
resultant charge density on each “plate” is then time-dependent because the flowing current causes
charge to pile up. At time zero there is no charge on the plates, so we know that the charge density
increases linearly as time progresses.

It
σ(t) = (2)
πa2
where a is the radius of the wire and It is the total charge on the plates at any instant (current is
in units of Coulombs/second, so the total charge on the end plate is the current multiplied by the
length of time over which the current has been flowing).

The electric field between the ends of the wire is,

~ = It
E ẑ (3)
o πa2

The magnetic field is found from Ampere’s law,


!
I Z ~
∂E
~ · d~l = µo Ienc + µo o
B · d~a Eq. 7.38 (4)
∂t

where the enclosed current is zero, Ienc = 0, and the magnetic field is generated entirely by the
displacement current.

1
The displacement current, Id (the non-zero term in Ampere’s law above), through the gap must be
in the ẑ direction (the only proper current flow is through the wire),

Bφ (2πs) = Id (5)

Solving for the displacement current in the gap gives,


!
Z ~
∂E
Id = µo o · d~a (6)
∂t
Z  
∂ It
= µo o ẑ · d~a (7)
∂t o πa2

µo I s 2π
Z Z
= (ẑ) · s ds dφ ẑ (8)
πa2 0 0

s
s2

µo I
= (2π) (9)
πa2 2 0

µo Is2
= (10)
a2

Now the magnetic field is found from (5),

~ = µo Is φ̂
B (11)
2πa2
where it should be noted that the field increases with s. This is only for the region in the gap and
inside the wire radius of a. The linear increase of the magnetic field in this region agrees with that
we would find in a solid wire with a uniform current density. For s > a the magnetic field is a
fringing field that goes to zero as s → ∞.

(b) The energy density is given in terms of the fields in the gap,

B2
 
1 2
uem = o E + Eq. 8.13 (12)
2 µo

I 2 t2 1 µ2o I 2 s2
 
1
= o 2 2 4 + (13)
2 o π a µo 4π 2 a4
 2  2
µo s2

1 I t
= + (14)
2 π 2 a4 o 4
~ · E.
noting that E 2 = E ~

The Poynting vector in the gap is,

~ = 1 ~ ~
S E×B Eq. 8.10 (15)
µo
 
1 It µo Is
= ẑ × φ̂ (16)
µo o πa2 2πa2

I 2 ts
= − ŝ (17)
2o π 2 a4

2
The Poynting vector represents energy flow. Taking special note of the direction found above we
see that the energy is flowing into the gap.

Verification of equation 8.14 follows (umech = 0 because there are no charges to move in the gap and
the energy within the gap is therefore due entirely to that of the fields),

∂ ~ ·S~
(umech + uem ) = −∇ Eq. 8.14 (18)
∂t
 2  2
µo s2 −I 2 ts
  
∂ I t 1 ∂
+ = − s (19)
∂t 2π 2 a4 o 4 s ∂s 2o π 2 a4

I 2t I 2 ts
 
1
= − − (20)
o π 2 a4 s o π 2 a4

I 2t
= (21)
o π 2 a4

(c) Since we have the energy density in the gap we are ready to determine the total energy.

B2
Z  
1 2
Uem = o E + dτ Eq. 8.5 (22)
2 µo

1 a 2π w
Z  2  2
µo s2
Z Z 
I t
= + s ds dφ dz (23)
2 0 0 0 π 2 a4 o 4
Z a 2
I2 t s µo s3

= (2πw) + ds (24)
2π 2 a4 0 o 4

I 2 w t2 a2 µo a4
 
= + (25)
πa4 2o 16

I 2 w t2 µo a2
 
= + (26)
2πa2 o 8

where this result is for the volume of the cylindrical gap (length w and radius a).

The problem tells us to determine the total power flowing into the gap by integrating the Poynting
vector over the surface enclosing it. This is the cylindrical surface occurring at s = a. Technically,
~ ·d~a =
this also includes the circular surfaces at each of the plates, but for these surfaces the product S
0 so they do not contribute to the solution.
Z 2π Z w
Power in = ~ · s dφ dz ŝ
S (27)
0 0
2π w
I 2 ts
Z Z
= − s dφ dz (28)
0 0 2o π 2 a4

I 2 ta2
= − (2wπ) (29)
2o π 2 a4

I 2 tw
= − (30)
o πa2

3
Finally, we essentially want to verify equation 8.9,
Z   I 
dW d 1 2 1 2 1 ~ ×B

~ · d~a
=− o E + B dτ − E (31)
dt dt V 2 µo µo S
where this represents the rate at which work is done on a collection of charges in the volume V, that
is enclosed by the surface S.

Since there are no charges in the gap, W = 0 and dW/dt = 0. The equation becomes,
Z   I 
d 1 2 1 2 1 ~ ×B

~ · d~a
o E + B dτ = − E (32)
dt V 2 µo µo S
I
d ~ · d~a
Uem = − S (33)
dt S

we have solved for Uem in (26), and the integral on the right hand side of the above equation is the
power flowing into the gap, given by (30). Continuing with the verification of equation 8.9,

d I 2 w t2 µo a2 I 2 tw
   
+ = − − (34)
dt 2πa2 o 8 o πa2

I 2 tw I 2 tw
= (35)
o πa2 o πa2
this equation is verified.

[2.] Professor Carter Problem


Two charged shells, both of radius R and with surface charge density σ, are placed so that there is
a distance d > 2R between their centers. Calculate the force of one shell on the other by integrating
Maxwell’s stress tensor. (Hint: a good surface to choose includes the plane which passes directly
between the two and then closes out at infinity).

Solution
Generally, the force on charges within a volume V is,
I Z
~
↔ d ~ dτ
F = T · d~a − o µo S Eq. 8.22 (36)
S dt V

In this problem we will determine the force on one of the spheres due to the other sphere using the
equation above. Enclose one of the spheres (call it number 2 for no specific reason) in a volume. In
this problem there is no time dependence. The term involving the time derivative of the Poynting

vector is zero. We will find the expression for the Maxwell stress tensor, T , which is determined by
the total field in the system (we sum the contributions from sphere 1 and sphere 2, recalling that the
stress tensor includes self-fields).

There are no magnetic fields in this problem so the Maxwell stress tensor is,
2
 
Ex2 − E2 Ex Ey Ex Ez
 
↔ 
2

T = o  Ey Ex Ey2 − E2 Ey Ez  (37)
 
 
 
E 2
Ez Ex Ez Ey 2
Ez − 2

4
The expression in (37) was given directly in lecture on 10/21/2005, but it may also be derived from
the general form of the stress tensor.
   
1 2 1 1 2
Tij ≡ o Ei Ej − δij E + Bi Bj − δij B Eq. 8.19 (38)
2 µo 2

I will draw the enclosed region around sphere 2 and determine the force that 1 exerts on it. The
electric fields outside of the spheres are determined from Gauss’ law. To best simplify the geometry
of the surface integral that must be done, I use a surface that includes the plane exactly between
the centers of the spheres. This surface closes at infinity, where the electric field is zero, so the
integration needs only be completed along the plane directly between the spheres (see figure 1).

Figure 1: Geometry of problem 2. The force is determined by integrating the Maxwell stress
tensor over a surface including the plane directly between the spheres.

Setting the origin of this system at the center of sphere 1, the electric field due to sphere 1 is given
by,

~1 = q
E r̂ (39)
4πo r2

σR2
= r̂ (40)
o r2

where q is the total charge on the sphere (therefore, q = σ(4πR2 )).

Writing this in terms of Cartesian coordinates, and setting x = d/2 since we are only interested in
the electric field along the plane between the charges gives,
!
2
 
~1 = σR 1 (d/2)x̂ + y ŷ + z ẑ
E (41)
(d/2)2 + y 2 + z 2
p
o (d/2)2 + y 2 + z 2
!
σR2
 
1 (d/2)x̂ + y ŷ + z ẑ
= (42)
(d2 /4) + y 2 + z 2
p
o (d2 /4) + y 2 + z 2

5
p
using r = (d/2)2 + y 2 + z 2 as the distance from the center of sphere 1 to the plane. The (d/2)x̂
term comes from my setting sphere 1 at the origin and placing center of the other sphere at x = d.
This is equivalent to treating the spheres as point charges located at their centers. Since the electric
field in question is that at the plane directly between them, this field could just as easily be due to
point charges as it is due to charged spherical shells.

Once again, stress tensor depends on the total field in the system. As such, the field of the second
sphere must be included. The symmetry in this problem allows the electric field due to the second
sphere to be written immediately after a translation in the x coordinate. The electric field of sphere
2 is the same as that due to sphere 1, except that x2 = x − d.
!
2
 
~2 = σR 1 (d/2 − d)x̂ + y ŷ + z ẑ
E (43)
(d/2 − d)2 + y 2 + z 2
p
o (d/2 − d)2 + y 2 + z 2
!
σR2
 
1 (−d/2)x̂ + y ŷ + z ẑ
= (44)
(−d/2)2 + y 2 + z 2
p
o (−d/2)2 + y 2 + z 2
!
σR2
 
1 (−d/2)x̂ + y ŷ + z ẑ
= (45)
(d2 /4) + y 2 + z 2
p
o (d2 /4) + y 2 + z 2

Add these to get the total electric field along the plane between them.

~ tot = E
E ~1 + E
~2 (46)

3/2
σR2

1
= [((d/2) − (d/2))x̂ + (y + y)ŷ + (z + z)ẑ] (47)
o (d2 /4) + y 2 + z 2
3/2
2σR2

1
= [y ŷ + z ẑ] (48)
o (d2 /4) + y 2 + z 2

This provides all the information needed to completely write out the stress tensor. First, note the
following,

Ex = 0 (49)

2σR2
 
y
Ey = (50)
o ((d /4) + y 2 + z 2 )3/2
2

2σR2
 
z
Ez = (51)
o ((d2 /4) + y 2 + z 2 )3/2
4σ 2 R4 y2 + z2
 
E2 = (52)
2o ((d2 /4) + y 2 + z 2 )3

Referring back to (36), the force on sphere 2 due to 1 may be written,


Z ∞Z ∞

~
F = T ·dy dz (−x̂) (53)
−∞ −∞

where the −x̂ represents the direction of the surface normal vector from the surface enclosing sphere
2.

6
In the following steps I have included the fact that Ex = 0 in order to simplify.
2
 
− E2 0 0
 
 −1

↔ 
2
T · (−x̂) = o  0 Ey2 − E2 Ey Ez   0  (54)
 
0
 
 
2 E2
0 Ez Ey Ez − 2
E2
 
= −o − x̂ + 0 ŷ + 0 ẑ (55)
2

o 4σ 2 R4 y2 + z2
 
= · x̂ (56)
2 2o ((d2 /4) + y 2 + z 2 )3

2σ 2 R4 y2 + z2
 
= x̂ (57)
o ((d2 /4) + y 2 + z 2 )3

Putting this back into the integral expression for the force provides,
Z ∞Z ∞
2σ 2 R4 y2 + z2
 
~
F = x̂ dy dz (58)
−∞ −∞ o ((d2 /4) + y 2 + z 2 )3

and it is seen that the final answer will put the force in either the positive or negative x̂. If this were
not the case, then we would already know our solution to be incorrect. Consider the top part of
sphere 1. It exerts a force on the bottom of sphere 2 that is directed partially along the −ẑ direction.
The bottom part of sphere 1, however, exerts a force on the top part of 2 that is directed along the
+ẑ direction and cancels out the previously mentioned −ẑ force. This symmetry is preserved for
the total force along the y and z directions, resulting in only a non-zero component along the x
direction. Furthermore, since the shells have the same surface charge density we know the force
between them should be repulsive. The force on 2 must be along the +x̂ direction or we know the
answer is incorrect.

The integral in (58) is non-trivial. Since there is no x dependence in the expression for the force along
the plane we can use change of coordinate system to obtain a betterpintegral with which to work.
Consider the yz plane to be in cylindrical coordinates. If we let s = y 2 + z 2 and then incorporate
the φ direction into the integral we get,
Z ∞Z ∞
2σ 2 R4 y2 + z2
 
~
F = x̂ dy dz (59)
−∞ −∞ o ((d2 /4) + y 2 + z 2 )3
Z ∞ Z 2π 2 4 
s2

2σ R
= x̂ s ds dφ (60)
0 0 o ((d2 /4) + s2 )3

where the new integral is the result of starting over with the geometry considerations and not a
mathematical change of variable. Notice that there is an extra factor of s that comes from the da
term in cylindrical coordinates. The x̂ dependence is left alone because it came from the tensor
work and the change to cylindrical coordinates was made after solving this part.

7
In this case the integral over s is solved using a change of variable and then an integral table.

s3
Z
ds → Let u = s2 du = 2s ds (61)
0 ((d2 /4)+ s2 )3
Z ∞
us du
= (62)
0 ((d2 /4) 3
+ u) 2s
Z ∞
1 u
= du (63)
2 0 ((d2 /4) + u)3
∞
(d2 /4)

1 u
= (−1) + (64)
2 (u + (d2 /4))2 2((d2 /4) + u)2 0

(d2 /4)
 
1
= − 0+0−0− (65)
2 2(d2 /4)2
1
= (66)
4(d2 /4)
1
= (67)
d2
The integral over φ results in a factor of 2π and the final solution is,

2σ 2 R4 1
F~ = (2π) 2 x̂ (68)
o d

4πσ 2 R4
= x̂ (69)
o d2

where you can use the expression q = 4πR2 σ to prove to yourself that this force is equivalent to that
between two like charged point particles separated by a distance d.

[3.] Problem by Professor Carter


Consider a cylindrical capacitor of length L with charge +Q on the inner cylinder of radius a and
−Q on the outer cylindrical shell of radius b. The capacitor is filled with a lossless dielectric with
dielectric constant equal to 1. The capacitor is located in a region with a uniform magnetic field B,
which points along the symmetry axis of the cylindrical capacitor. A flaw develops in the dielectric
insulator, and a current flow develops between the two plates of the capacitor. Because of the mag-
netic field, this current flow results in a torque on the capacitor, which begins to rotate. After the
capacitor is fully discharged (total charge on both plates is now zero), what is the magnitude and
direction of the angular velocity of the capacitor? The moment of inertia of the capacitor (about the
axis of symmetry) is I, and you may ignore fringing fields in the calculation.

Reference the figure below.


Solution
Use the concept of the conservation of angular momentum to solve this problem quickly. Before
the current flow there is a certain amount of angular momentum stored in the EM fields. After the
dielectric breaks down, the capacitor discharges until there is no longer an electric field between its
plates. Since there is no longer an electric field there is no longer any angular momentum stored in
the fields. All of this angular momentum must now exist in the physical rotation of the system.

8
+Q

-Q

It is still possible to solve this problem considering the force on the current, but that would require
considerably more work than the method shown here. Furthermore, the problem statement doesn’t
say much about the current; is it uniform or localized?

The total angular momentum in the fields is,


Z h i
~ em = o ~r × (E
L ~ × B)
~ dτ (70)

where the integrand is the angular momentum density given as equation 8.34 in Griffiths.

The magnetic field is given in the problem. For a cylindrical capacitor the electric field is known to
be zero outside the plates. Inside the capacitor we have (a result determined by applying Gauss’
law to this system, the full solution of which is written out elsewhere),

~ = λ
E ŝ (71)
2πs
where λ is the charge per unit length of the inner cylinder and  is the dielectric constant of the
material between the plates. We are given that  = 1.

The term E ~ ×B ~ = 0 everywhere except the region between the plates. The limits of the volume
integral in (70) are then decided. Continuing with the solution for the total angular momentum
stored in the fields gives,
Z bZ 2π Z L  
~ em = λ
L ~s × ŝ × B ẑ s ds dφ dz (72)
a 0 0 2πs
Z b Z 2π Z L  
λB
= s ŝ × (−φ̂) s ds dφ dz (73)
a 0 0 2πs
Z b Z 2π Z L
λBs
= − ds dφ dz ẑ (74)
a 0 0 2π
Z b
λB
= − (2π)(L) s ds ẑ (75)
2π a

9
λB 2
= − (b − a2 )L ẑ (76)
2
QB 2
= − (b − a2 ) ẑ (77)
2
where the total charge on the inner cylinder is Q = λL.

This is the total angular momentum in the system. When the electric field between the plates of
the capacitor goes to zero this angular momentum will be entirely contained within the physical
~ = I~
rotation of the system. Angular momentum is related to angular velocity as L ω , where I is the
moment of inertia. The solution is,
~
L QB 2
ω
~ = = − (b − a2 ) ẑ (78)
I 2I
where this provides both the direction and magnitude of the angular velocity. The cylindrical ca-
pacitor will be spinning about its axis as electromagnetic angular momentum is converted to kinetic
angular momentum.

[4.] Problem 8.9 from Griffiths


Consider a very long solenoid. This solenoid has radius a, turns per unit length n, and a current
Is flowing through it. A loop of wire with resistance R is coaxial with the solenoid. The radius of
this wire loop, b, is much greater than the radius of the solenoid. The current in the solenoid is then
slowly decreased, leading to a current flow, Ir , in the loop.

(a) Find Ir in terms of dIs /dt.

(b) The energy dissipated in the resistor must come from the solenoid. Calculate the Poynting vector
just outside the solenoid and verify that it is directed toward the loop. (Griffiths Hint: Use the
electric field due to the changing flux in the solenoid and the magnetic field due to the current in
the wire loop.) Integrate the Poynting vector over the entire surface of the solenoid to verify that the
total energy “emitted” by the solenoid is equal to that dissipated in the resistive wire, Ir2 R.

Solution
(a) This part is a review of the topics covered in Ch. 7 of Griffiths. The current through a resistive
wire is Ir = E/R, where E is the emf (voltage) across the wire. The emf is calculated according to,

E = − (79)
dt
d
= − µo nIs πa2 (80)
dt
dIs
= −µo nπa2 (81)
dt
where this takes advantage of the properties of solenoid fields (i.e. the field outside is zero and the
field inside is uniform).

The current in the loop is positive, though the direction depends on how we orient the solenoid, let
~ → ẑ.
B
µo nπa2 dIs

Ir = (82)
R dt

10
The absolute value of the time derivative term is taken because we know the current in the solenoid
is decreasing.

(b) Begin by solving for the fields Griffiths tells us to use. The electric field at the surface of the
~ ×E
solenoid is found using Faraday’s law, ∇ ~ = −dB/dt.
~ Since we care about the field at the surface
of the solenoid we set s = a.
I Z
~ ~ d ~ · d~a
E · dl = − B (83)
dt

Eφ (2πa) = − (84)
dt

~ = −µo nπa2 dIs 1 φ̂


E (85)
dt 2πa

µo na dIs
= φ̂ (86)
2 dt

Notice that the final direction of the electric field is in the positive φ̂. There is a negative sign in
Faraday’s law, but we also know that the time derivative of the solenoid current is negative since it
is being slowly decreased. We could also use Lenz’s law to determine this direction: the magnetic
field of the solenoid is decreasing (and we said that B ~ = B ẑ), so the current induced in the wire
will be such as to try and replace this decreasing field. A wire generates a magnetic field in the +ẑ
direction with a current in the +φ̂ direction.

The magnetic field at the surface of the solenoid is greatly simplified since b  a. This means we
may treat the entire surface as though it lies along the z-axis of the loop. The magnetic field along
the z-axis of a current loop is given in example 5.6 of Griffiths,

~ µo Ir b2
B(z) = ẑ Eq. 5.38 (87)
2 (b2 + z 2 )3/2
where the direction is set by the orientation of the solenoid.

The Poynting vector may now be calculated,

b2

~ 1 ~ ~ = 1 µo na dIs µo Ir
S = E×B φ̂ × ẑ (88)
µo µo 2 dt 2 (b2 + z 2 )3/2

µo nab2 Ir dIs

= ŝ (89)
4(b2 + z 2 )3/2 dt
The Poynting vector is directed toward the resistive loop, as expected. The next step is to integrate
this vector over the entire surface of the solenoid (still using s = a).
Z 2π Z ∞ 
µo nab2 Ir dIs
Z 
P ower = ~
S · d~a = ŝ · a dφ dz ŝ (90)
2 2 3/2 dt
0 −∞ 4(b + z )
Z ∞
µo na2 b2 Ir dIs

dz
= (2π) 2 + z 2 )3/2
(91)
4 dt
−∞ (b

From integral tables, Z


dx x
= (92)
(f + cx2 )3/2 f (f + gx2 )1/2

11
∞
µo πna2 b2 Ir

dIs z
P ower =
dt b2 (b2 + z 2 )1/2 (93)
2 −∞

µo πna2 b2 Ir dIs 2
 
= dt b2 (94)
2

2 dIs

= µo πna Ir (95)
dt

Using (82) to rewrite (95),


P ower = Ir2 R (96)
and the power directed from the solenoid toward the resistive loop is equal to the energy dissipated
in this loop.

[5.] Problem 8.11 from Griffiths


Treat the electron as a uniformly charged spherical shell (total charge e) of radius R, spinning with
angular velocity ω.

(a) Determine the total energy contained in the EM fields.

(b) Find the total angular momentum contained in the EM fields.

(c) Let the mass of the electron be described completely in terms of the energy stored in its fields,
Uem = me c2 . Furthermore, let the spin angular momentum of the electron be due entirely to its
fields, Lem = ~/2. Solve for the angular velocity and radius of the electron in this case. Calculate
the value Rω and comment on whether this value makes sense classically.

Solution
(a) The total energy in the fields is given by (22). This is the sum of the energy in the electric field
plus that of the magnetic field, so I will determine Ue and Um independently. Begin by determining
the fields inside and outside the shell.

From electrostatics and Ch. 2 of Griffiths we know that the electric field inside a uniformly charged
spherical shell is zero. Also, the field outside is that of a point charge. Therefore,

~ in = 0
E (97)

~ out = e
E r̂ (98)
4πo r2

The magnetic field inside a uniformly charged spinning spherical shell is given in example 5.11 of
Griffiths,
~ in = 2 µo σRω ẑ
B Eq. 5.68 (99)
3
This can be rewritten in terms of the variables given in the problem after we solve for the surface
charge density,
Qtot e
σ = = (100)
A 4πR2

12
The magnetic field inside the shell is,
~ in = µo ωe ẑ
B (101)
6πR
The magnetic field outside of the shell is that of a dipole. This is known from a variety of sources:
Your work in PHYS 110A, reading problem 5.36 from Griffiths, or using the given vector potential
from example 5.11 in Griffiths to solve for the field directly by way of B~ =∇ ~ × A.
~ Regardless of
your method, the magnetic field outside the shell is,

~ out = µo m 2 cos θ r̂ + sin θ θ̂


 
B (102)
4πr3

µo ωeR2  
= 2 cos θ r̂ + sin θ θ̂ (103)
12πr3
I have included the value of the dipole moment in (103) because you probably solved for it in 110A
and referenced that solution to provide its value here. If you do not remember this value, then you
might have solved for it in the following way.

Break the spinning shell into a series of infinitesimal rings. In this case the differential element of
the magnetic dipole moment is given by,

dm
~ = dI ~a (104)

The differential element on the right hand side of (104) must be for the current because the area
vector of any individual ring is,

~a = πl2 ẑ (105)

= π(r sin θ)2 ẑ (106)

= πr2 sin2 θ ẑ (107)

where l is the radius of the ring. The direction ẑ is determined by the orientation of the sphere and
its rotation and can therefore be set to whatever value we want.

To find dI we need to write out the current through an individual ring. This is determined using
the surface current density.

dI = K dL where dL is the θ̂ component of the spherical length dl

= σv(r dθ)

ω × ~r|r dθ
= σ|~

= σωr2 sin θ dθ

Returning to (104),
~ = πσωR4 sin3 θ dθ
dm (108)
where I have taken into account the fact that this is a shell so r = R.

13
The magnetic dipole moment is found through an integration of dm,
~
Z π
m~ = πσωR4 sin3 θ dθ ẑ (109)
0

 π
 
1 4 2
= πσωR − cos θ sin θ + 2 ẑ (110)
3 0

4πσωR4
= ẑ (111)
3
~ out .
Replace the charge density in (111) with that from (100) to get the final expression for B

Now begins the calculation of the total energy in the fields. The electric field is zero inside the shell
so it contributes nothing to the total energy. The electric field outside the shell contributes,
o E 2
Z
Ue,out = dτ (112)
2
2
o ∞ π 2π
Z Z Z 
e
= r2 sin θ dr dθ dφ (113)
2 R 0 0 4πo r2
Z ∞
e2 dr
= 2
(4π) 2
(114)
32π o R r
 ∞
e2 1
= − (115)
8πo r R

e2
= (116)
8πo R
On to the magnetic field energy inside the shell. Since the field inside the shell is uniform the integral
may be skipped. The energy inside the shell is simply the magnetic energy density multiplied by
the interior volume.
4
Um,in = um,in · πR3 (117)
3
1  µo ωe 2 4 3
= · πR (118)
2µo 6πR 3

µo ω 2 e2 R
= (119)
54π
Calculating the energy stored in the magnetic field outside of the shell will illustrate why I wrote
the dipole field in terms of spherical coordinates. Once again, recall that B 2 = B ~ · B.
~
Z ∞ Z π Z 2π  2 2 2 4
1 µo ω e R
Um,out = (4 cos2 θ + sin2 θ)r2 sin θ dr dθ dφ (120)
R 0 0 2µo 144π 2 r6
Z ∞Z π
µo ω 2 e2 R4 1
= 2
(2π) 4
(4 cos2 θ + sin2 θ) sin θ dr dθ (121)
288π R 0 r

µo ω 2 e2 R4 1 ∞ π
  Z
= − 3 (4 cos2 θ + sin2 θ) sin θ dθ (122)
144π 3r R 0

µo ω 2 e2 R π
Z
= (4 cos2 θ + sin2 θ) sin θ dθ (123)
432π 0

14
This is another integral that you may solve in any manner. Here I make the substitution 4 cos2 θ =
4 − 4 sin2 θ. The integral then reduces to,
Z π Z π
(4 cos2 θ + sin2 θ) sin θ dθ = (4 − 3 sin2 θ) sin θ dθ (124)
0 0
Z π
= 4(2) − 3 sin3 θ dθ (125)
0

 π
 
1 2
= 8 − 3 − cos θ sin θ + 2 (126)
3 0

= 8−4 = 4 (127)

Returning to the energy expression we have,

µo ω 2 e2 R
Um,out = ·4 (128)
432π

µo ω 2 e2 R
= (129)
108π

The total energy in the fields is the sum of (116), (119), and (129).

e2 µo ω 2 e2 R µo ω 2 e2 R
Utot = + + (130)
8πo R 54π 108π

e2 µo ω 2 e2 R
= + (131)
8πo R 36π

(b) The angular momentum stored in the fields is found using (70). The zero electric field inside the
shell means that we only need be concerned with the region outside of the shell. All of the fields
have already been found, so we begin by calculating the angular momentum density in the fields
and then we’ll integrate that over the volume outside of the shell.
h i
~lem = o ~r × (E ~ out × B
~ out ) (132)

µo ωeR2 
  
e
= o ~r × r̂ × 2 cos θ r̂ + sin θ θ̂ (133)
4πo r2 12πr3

µo ωeR2
  
e
= o ~r × · sin θ φ̂ (134)
4πo r2 12πr3

µo ωe2 R2
= sin θ (−r θ̂) (135)
48π 2 r5

µo ωe2 R2
= − sin θ θ̂ (136)
48π 2 r4

15
On to the total,
∞Z π 2π
µo ωe2 R2
Z Z
~ em =
L −sin θ θ̂ r2 sin θ dr dθ dφ (137)
48π 2 r4
R 0 0
Z ∞Z π
µo ωe2 R2 sin2 θ
= − (2π) dr dθ θ̂ (138)
48π 2 R 0 r2
 ∞ Z π
µo ωe2 R2 1
= − − sin2 θ dθ θ̂ (139)
24π r R 0

µo ωe2 R π 2
Z
= − sin θ dθ θ̂ (140)
24π 0

The integral in (140) is not as easy as it looks. The θ̂ vector changes with the value of θ. Integrating
over θ from 0 to π means the θ̂ can be replaced with its z component. This is a mathematical fact
and you will be able to find it elsewhere (proven rigorously). The z component of θ̂ is (− sin θ).
π
µo ωe2 R µo ωe2 R π 2
Z Z
2
− sin θ dθ θ̂ = − sin θ dθ (− sin θ ẑ) (141)
24π 0 24π 0

µo ωe2 R π 3
Z
= sin θ dθ ẑ (142)
24π 0

The solution to the integral is shown in (125) and (126).


2
~ em = µo ωe R ẑ
L (143)
18π

(c) We can use the angular momentum relation to solve for the product, ωR, immediately.

~
Lem = (144)
2

µo ωe2 R ~
= (145)
18π 2
18π~
ωR = (146)
2µo e2

9π(1.05 × 10−34 )
= (147)
(4π × 10−7 )(1.60 × 10−19 )2

= 9.23 × 1010 (148)

This product represents the physical speed (in m/s) of a point on the equator of the shell. It is
considerably faster than the speed of light and therefore makes no sense physically, demonstrating
the need for quantum mechanics in the explanation of various properties of the electron.

To solve for the values independently use (148) and the mass relation given in the problem state-
ment, Uem = me c2 . This provides two equations with which you can solve for the two unknowns.

16
The numerical values are approximately:

R = 2.96 × 10−11 m (149)

ω = 3.12 × 1021 s−1 (150)

[6.] Problem 9.6 from Griffiths


(a) Write a revised boundary condition (replacing equation 9.27 from Griffiths) for the case of a
tension T applied across two strings connected with a knot of mass m.

(b) Consider the situation where the knot connecting the strings has mass m and the second string
is massless. Find the amplitudes and phases of the reflected and transmitted waves.

Solution
(a) Begin with,
∂f ∂f
= Eq. 9.27 (151)
∂z 0− ∂z 0+
where the + and − subscripts refer to the right and left sides of the knot respectively.

To determine the new boundary condition, refer to the origin of Eq. 9.27 (page 365 of Griffiths),
 
∂f ∂f
∆F ∼

=T − (152)
∂z + ∂z −

The above equation is considered at the point z = 0, the location of the knot. Griffiths arrives at
equation 9.27 by taking the left side of (152) to be zero because the mass of the knot is set to zero.
In part (a) of this problem we are asked to consider a knot of some mass, m. This is equivalent to
setting ∆F = ma, which for a one dimensional case becomes,

∂2f
 
∂f ∂f
m 2 =T − (153)
∂t ∂z + ∂z −

all of which is evaluated at z = 0. Recall that the function describing the string, f , represents its
position. The derivative of f is the velocity, and the second derivative is an acceleration.

(b) This part is a boundary value problem. We will use two equations to solve for the amplitudes and
phases of the reflected and transmitted waves in terms of the incident values (which may always be
assumed to be given). The first boundary condition is given in (153). The second boundary condi-
tion comes from the fact that the rope itself is continuous and therefore requires that the functions
to the left and right of the knot be equal at z = 0.

f (0− , t) = f (0+ , t) Eq. 9.26 (154)

The general solution is already known from the properties of waves.


∼ ∼
f− = AI ei(k1 z−ωt) + AR ei(−k1 z−ωt) (155)


f+ = AT ei(k2 z−ωt) (156)

17
∼ ∼ ∼
where AI , AR , and AT refer to the complex amplitudes of the incident (coming from the left), re-
flected, and transmitted waves respectively.

Condition (154) says that we can use either wave function for the time derivative term in (153).
Using f+ we have,

∂2f ∼
m = −mω 2 AT e−iωt (157)
∂t2
recalling that this is evaluated at z = 0. The shortcut method is to use ∂/∂t = −iω when dealing
with waves of this sort.

Computing the right side of (153),


∼ ∼ ∼ ∼
−mω 2 AT e−iωt = iT (k2 AT e−iωt − k1 AI e−iωt + k1 AR e−iωt ) (158)

∼ ∼ ∼ ∼
−mω 2 AT = iT (k2 AT −k1 AI +k1 AR ) (159)

This allows the transmitted amplitude to be written in terms of the other amplitudes as,
∼ ∼ ∼
(iT k2 + mω 2 ) AT = iT k1 (AI − AR ) (160)

Writing out (154) allows us to simplify it,


∼ ∼ ∼
AI + AR = AT (161)

Multiply (161) by iT k1 and add this to (160),


∼ ∼ ∼
iT k1 · (AI + AR = AT ) (162)

∼ ∼ ∼
(iT k2 + mω 2 ) AT = iT k1 (AI − AR ) (163)

Result :

∼  ∼
iT (k1 + k2 ) + mω 2 AT

2iT k1 AI = (164)

∼ 2iT k1 ∼
AT = 2
AI (165)
[iT (k1 + k2 ) + mω ]
∼ ∼
Putting this expression for AT into (161) and solving for AR gives,
∼ iT (k1 − k2 ) − mω 2 ∼
AR = AI (166)
iT (k1 + k2 ) + mω 2
Now it is time to make use of the fact that the second string is massless. For waves on strings the
velocity is given by s
T
v= Eq. 9.3 (167)
µ

18
where µ is the mass density of the string.

In this problem µ2 = 0 so v2 = ∞. The wave vectors of the two waves are related to the velocities
by,
k2 v1
= Eq. 9.24 (168)
k1 v2
which leads to k2 /k1 = 0 in this case.

Return to the expressions for the transmitted and reflected amplitudes in terms of the incident am-
plitude and factor k1 out of the denominators. This allows those expressions to simplify to,
∼ 2 ∼
AT = imω 2
AI (169)
1− k1 T
imω 2
∼ 1+ k1 T

AR = imω 2
AI (170)
1− k1 T

Separate the real amplitude from the phase of the waves as follows,
2
AT eiδT = imω 2
AI eiδI (171)
1− k1 T
imω 2
iδR
1+ k1 T
AR e = imω 2
AI eiδI (172)
1− k1 T

Thus concludes the setup part of this problem. From this point forward it is all algebra. One method
is the following,
AT eiδT 2
= imω 2
(173)
AI eiδI 1− k1 T
imω 2
AT eiδT 1+ k1 T
= imω 2
(174)
AR eiδR 1− k1 T

If you square both sides of the above equations you will be able to separate out the ratio of the
amplitudes from a relation between the phases. Coupling this with the following identity,
ei2φ − 1
tan φ = (175)
i (ei2φ + 1)
will allow you to solve for the desired values in terms of the incident parameters.

After some algebra, the real amplitudes and phases are,


2
AT = q AI (176)
m2 ω 4
1+ k12 T 2

AR = AI (177)

mω 2
δT = δI + tan−1 (178)
k1 T
2mω 2
−1 k1 T
δR = δI + tan 2 ω4 (179)
1 −m k12 T 2

19
[7.] Problem by Professor Carter
A clever cub scout who has studied E&M decides he wants to cheat in the Pinewood derby race
by giving his car an extra push using a laser. He happens to have one of the most powerful CW
(continuous wave, meaning it can lase continuously) lasers available, with a laser power of 10 kW.
He plans to shine this laser on the back of the car, which is coated with a perfect reflector. To figure
out is his scheme will help him, calculate the length of time necessary for the laser to accelerate his
0.1 kg car to a speed of 1 m/s. (Note that if the reflector isn’t perfect, then the car will be destroyed
long before reaching this speed.)

Solution
This is a problem in kinetics. The laser will exert a force on the car that is described according to
the radiation pressure of the laser. Since the laser is on continuously, the force on the car will be
constant. As such, the car’s acceleration due to the laser will also be constant. The time dependence
of an object’s velocity due to a constant acceleration (a) is,

v(t) = vo + at (180)

where vo = 0 in this problem (not explicity stated in the problem, but since this is a race the car must
start from rest). The final velocity is v = 1.

Pressure, which is force (F ) per unit area (A), is given by,


F 2I
P = = Eq. 9.64 (181)
A c
where I is the intensity of the incident light and we need to rewrite this to solve for the force on the
car due to the laser. The factor of 2, which is not actually written in Griffiths’ form of the equation,

F = PA (182)

2IA
= (183)
c
Intensity is defined as the average power per unit area (see page 381 in Griffiths). The laser power
(PL ) is given in the problem, which brings us to,
 
PL 2A
F = (184)
A c
2PL
= (185)
c

2 × 104
= (186)
3 × 108

ma = 0.66 × 10−4 (187)

0.66 × 10−4
a = (188)
0.1

= 0.66 × 10−3 (189)

20
Now we can solve for the time it takes to reach the velocity of 1 m/s.

v = at (190)

1 = 0.66 × 10−3 t (191)

t = 1500 (192)

where this value is in units of seconds. This is equivalent to 25 minutes, meaning that it is unlikely
the scout will derive any benefit from this laser propulsion system.

[8.] Problem by Professor Carter


Calculate ∇ ~ ·S
~ and ∂Uem /∂t for a linearly polarized electromagnetic plane wave propagating in the
ẑ direction and polarized in the x̂ direction. Explain your results physically.

Solution
The ”paradigm” for describing such an EM wave is (Eq. 9.48),

~ t) = Eo cos(kz − ωt + δ) x̂
E(z, (193)

~ t) = 1
B(z, Eo cos(kz − ωt + δ) ŷ (194)
c

where the direction of propagation for this wave is given according to k̂ = Ê × B̂ = ẑ

~ ·S
The term ∇ ~ is,

~ ·S
∇ ~ · 1E
~ = ∇ ~ ×B
~ (195)
µo
2
~ · Eo cos2 (kz − ωt + δ) ẑ
= ∇ (196)
µo c

Eo2 ∂
= cos2 (kz − ωt + δ) (197)
µo c ∂z

−2kEo2
= cos(kz − ωt + δ) sin(kz − ωt + δ) (198)
µo c

The term ∂Uem is,


∂Uem ∂ 
o E 2

= (199)
∂t ∂t
∂ 
o Eo2 cos2 (kz − ωt + δ)

= (200)
∂t

= o Eo2 (2ω cos(kz − ωt + δ) sin(kz − ωt + δ)) (201)

= 2o ωEo2 cos(kz − ωt + δ) sin(kz − ωt + δ) (202)

21
Determine the physical meaning of these results by comparing the results in (198) and (202),
−2kEo2
~ ·S
∇ ~ µo c cos(kz − ωt + δ) sin(kz − ωt + δ)
∂Uem
= (203)
∂t
2o ωEo2 cos(kz − ωt + δ) sin(kz − ωt + δ)
−k
= (204)
µo o ωc
Now we must recall the following expressions relating k, ω, and the speed of light.
ω
= c (205)
k
1
= c2 (206)
µo o
Continuing on,
~ ·S
∇ ~ −c2
∂Uem
= (207)
∂t
c2

= −1 (208)

These terms are opposite in magnitude. As the energy in an EM wave decreases in time, the amount
of energy flux away from the present location increases. This is another demonstration of energy
conservation: whatever is lost within a system must have been radiated away. The topic of radiation
arrives soon.

[9.] Problem 9.12 from Griffiths


Find the elements of the Maxwell stress tensor for a monochromatic plane wave (same one from the
previous problem, given by (193) and (194)). Is this the answer you expected? How is the Maxwell
stress tensor related to the energy density?

Solution
The Maxwell stress tensor is given by (38). In this problem there are only Ex and By components of
Tij . Most of the components can be immediately seen to be zero. The remaining terms are,
  2

B2

o Ex2 − E2 − 2µ o
0 0
 
 
↔  
2
 2 2

T = cos (kz − ωt + δ)   E 1 2 B  (209)
 0 − 2 + µo By − 2
o
0 
 
 
o E 2 B2
0 0 − 2 − 2µo

where I have factored out the common cosine term.

In this problem Ex2 = E 2 and By2 = B 2 , which further simplifies this expression to,
o E 2 B2
 
2 − 2µo 0 0
 
↔ 
2

2 B2
T = cos (kz − ωt + δ)  0 − o2E + 0 (210)
 
 2µo 

 
2 B2
0 0 − o2E − 2µo

22
Now recall from the previous work on electromagnetic energy that (see Eq. 9.54 for a reminder),

o E 2 B2
= (211)
2 2µo

This makes two more of the terms in (210) zero,


 
0 0 0
 
↔  
T = cos2 (kz − ωt + δ)  0 0 0 (212)
 

 
 
2 B2
0 0 − o2E − 2µo

The Maxwell stress tensor is the negative of the energy density. For EM waves (light waves) the
energy density is completely manifested in the momentum density.

23

You might also like